Physics 15 Torque Example 3 (3 of 7) Mass on Rod and Cable

Поделиться
HTML-код
  • Опубликовано: 6 сен 2024
  • Visit ilectureonline.com for more math and science lectures!
    In this third of the seven part series I will show you how to find the tension of a cable attached to a wall and rod with a mass hanging at the end of the rod.

Комментарии • 185

  • @matrixate
    @matrixate 9 лет назад +113

    Yes, the 1/2 was over-looked...but what really matters is knowing how to solve this problems. Great video!
    Anyway, the correct final answer is supposed to be T=120483.4406N or just 1.2x10^5N

    • @tenzintophel7179
      @tenzintophel7179 5 лет назад +3

      he got it wrong, I was confused for a bit but he saved me from my physics test

    • @neohlitana4878
      @neohlitana4878 3 года назад +2

      No, it's 12048.244

    • @drtao8325
      @drtao8325 Год назад +1

      yes i agree, he made calcultion error

  • @MichelvanBiezen
    @MichelvanBiezen  10 лет назад +58

    Anuj. As some students before, you found the error with the 1/2 factor. Very good!

    • @ceit.estella
      @ceit.estella 3 года назад

      sir Michel van Biezen, statics problems next please. Thanks.

  • @MichelvanBiezen
    @MichelvanBiezen  9 лет назад +9

    Felipe
    You are correct. That is a mistake.

  • @TheBuzz187
    @TheBuzz187 9 лет назад +5

    I'm still following right along with you MVB! You have provided lectures for EVERY single section we have covered so far in my phys-1 course! You are a God send! I truly thank you (again), with the utmost sincerity! You explain things in such a way that I can comprehend, and that takes talent! Well done, sir.

  • @chellychocolate285
    @chellychocolate285 10 лет назад +3

    Your saving my life right now! God bless you immensely!

  • @munnesa1
    @munnesa1 7 лет назад +2

    He has made my physics semesters easier and more clear. Appreciate your help!

  • @1junebugonly
    @1junebugonly 9 лет назад +1

    I hope you read this, your videos are f****** amazing! They helped me save valuable time and dramatic anxiety.

    • @MichelvanBiezen
      @MichelvanBiezen  9 лет назад

      harold alvarez
      Harold,
      I do try to keep up to the comments. Much appreciated.

  • @SilvaGalaxy9537
    @SilvaGalaxy9537 2 года назад +2

    strategy works in all examples.thank you so much for the most productive videos..Love from South Africa

  • @puretwr899
    @puretwr899 3 года назад +2

    You are the best physic teacher ever👍

  • @jacobthomas6908
    @jacobthomas6908 8 лет назад +6

    Professor Biezen, thank you so much for these videos! :) Your explanations are succinct and sweet, just how physics should be :)

  • @tylerrejko9492
    @tylerrejko9492 6 лет назад +1

    These videos are great. Very helpful in understanding multiple physics concepts

  • @drujjawalrathore
    @drujjawalrathore 6 лет назад

    I'm glad that I scrolled once down to look at the comment section. I was just going to try that once again (after already working it up twice and getting the tension as 1.2*10^5.)

  • @Steve76581
    @Steve76581 9 лет назад +1

    This video definitely simplifies things. Great teaching methods.
    Tks

  • @Egbe21
    @Egbe21 Год назад +2

    I’m definitely dedicating this engineering degree to you sir😹😹😹 you’re a life saver

    • @MichelvanBiezen
      @MichelvanBiezen  Год назад +1

      Glad you found our videos! Keep it going! 🙂

  • @juju6351
    @juju6351 7 лет назад +2

    Thank you so much, you saved my life! Literally

  • @clee888
    @clee888 9 лет назад

    My mom was listening to me watch this as I study for the MCAT and her English is not that good, but she said "when teachers explain things this clearly, how can you not understand?." She then continued, "he sounds as charming as David Attenborough," who is considered a treasure of England, and a famous naturalist.

    • @MichelvanBiezen
      @MichelvanBiezen  9 лет назад

      Binh,
      Thank you for the compliment. I really like David Attenborough. I wish I could present material as well as he.

  • @Peter_1986
    @Peter_1986 4 года назад +1

    I have noticed that most people seem to prefer clockwise rotation as the positive rotation over anticlockwise rotation - I guess it's probably because the sun and the hands on a watch move clockwise, so it feels more natural and intuitive to use that movement as the positive rotation for those reasons.

    • @MichelvanBiezen
      @MichelvanBiezen  4 года назад

      I do as well. But when we use torque as a vector, the convention is that counterclockwise is positive.

  • @geojohn5299
    @geojohn5299 9 лет назад +18

    The answer is 1.2 X 10e5 N.

    • @TrailBlazer65
      @TrailBlazer65 8 лет назад +2

      +Geo John I keep getting that too! The only thing I did differently to him here was not cancel out the Length.

    • @bloodassassinchild
      @bloodassassinchild 8 лет назад +2

      +Geo John You are correct, he forgot to keep the 1/2 after canceling all the Ls from the equation.

    • @TopAhmed1
      @TopAhmed1 8 лет назад +1

      120606.3829 ........

  • @naviibbyy07
    @naviibbyy07 10 лет назад +5

    why can't you be my professor?? love your videos

  • @abdou290
    @abdou290 9 лет назад +2

    Thanks for the nice problem. You forgot to mention the pivot force on the beam. It got cancelled because you took the torque about the pivot point. Also you forget to divide Mg by 2. But anyway, it was nice all the way.

  • @footlongchen
    @footlongchen 6 лет назад +2

    Amazing and clear example, thank you!

  • @pankaj37baghel
    @pankaj37baghel 3 года назад +1

    Why is normal reaction from the point string makes angle of 30 is not considered?

  • @pipertripp
    @pipertripp 6 лет назад +1

    Fun problem. Thx for putting all of these together.

  • @psilvakimo
    @psilvakimo 5 лет назад +1

    A free-body-diagram with a coordinate system will make the problem mach easier to explain.

  • @fapalisok97
    @fapalisok97 10 лет назад

    Ap physics on the line!!!! Self studying sucks though no other kids to do labs with

  • @ironuranium3927
    @ironuranium3927 5 лет назад +1

    Everything static in here so summation of all forces in the y-direction equal to zero (2000*9.8+5000*9.8=Tsin(30) ) but the result comes different which is wrong in the equation?

  • @BearfootBrad
    @BearfootBrad Год назад +1

    I freaked when my answer was different but happy when I saw I caught the overlooked 1/2

  • @abdullahahmethan2890
    @abdullahahmethan2890 10 лет назад +12

    the answer is 1.205*10^5

  • @samsonridge4526
    @samsonridge4526 4 года назад +1

    you made an error sir.Just one.YOu forgot to use the 1/2.But you are the greatese sir

  • @dariosilva85
    @dariosilva85 2 года назад +2

    You forgot a factor of 1/2 for the torque coming from the big M.

  • @MURTYPHYSICSVIDEOS
    @MURTYPHYSICSVIDEOS 8 лет назад

    Good morning Mr. Michel van Biezen
    Your lexplanation is very good .
    I like your videos.
    In this video d2=L/2Cos45°
    Please look at the video 8:18 min. You have ignored denominator 2

  • @karenalejandravasquez8885
    @karenalejandravasquez8885 7 лет назад +1

    Thank you so much! You have no idea how helpful you have been to me! Much respect.

  • @anilkumarsharma1205
    @anilkumarsharma1205 5 лет назад +1

    do you know about tachometer reading formula according to which it's reading increases
    I see that it's reading difference hours meter time according to your wrist watch are different by a variety of different fraction to the real time according to your wrist watch???
    please tell about that exact fraction according to which watch time of tachometer and wrist watch time are in ratio, forever right😌😌😌😌😌

  • @tongmac-harrison
    @tongmac-harrison Год назад +1

    please sir i don't understand where you kept the half attached to Mgcos teta

    • @MichelvanBiezen
      @MichelvanBiezen  Год назад +1

      That was an error. You found that error, good for you.

  • @LearnWithFardin
    @LearnWithFardin 2 года назад +1

    A Handsome guy (sir)!💝🇧🇩

  • @deNNyTheWiseMAN1
    @deNNyTheWiseMAN1 7 лет назад +2

    I think the 1/2 was overlooked, even the experts will make a mistake every now and then.

    • @MichelvanBiezen
      @MichelvanBiezen  7 лет назад

      Yes, you found it as well. See the comments below.

  • @johnperry3093
    @johnperry3093 5 лет назад +2

    Thank you. This helped a lot.

  • @amilaprasanga9763
    @amilaprasanga9763 8 лет назад +1

    it was a very useful video thank u so much. unfortunately missed that 1/2, anyway it was very helpfull

  • @nildolabtic6990
    @nildolabtic6990 9 лет назад

    yes if u cancel L where is the 2... the other,, d sub 3 maybe its not only a sin of 15 deg.but instead it is sin15(10)....but never mind the explanation and effort are pretty good thank you Sir, its remind me the past..... but until now i'm still learning.....TY

  • @abdun-nurkadil465
    @abdun-nurkadil465 3 года назад +1

    you really helped me there sir.

  • @anilkumarsharma1205
    @anilkumarsharma1205 5 лет назад +1

    is their any app regarding this formula for physics of crane

  • @ManishVerma-qe6cw
    @ManishVerma-qe6cw 6 лет назад +2

    Sir v.good video for learning.
    Sir how can I found Mg weight if we know the weight of L

  • @seer9293
    @seer9293 7 лет назад +1

    OML.You are awesome.subscribed!I just wish these videos were new so that I could thank you now..... :D

  • @oliviasetwaba8869
    @oliviasetwaba8869 6 лет назад +1

    Do you know how to determine forces graphically, especially forces on the sheeleg crane

  • @valcheung5747
    @valcheung5747 3 года назад

    how come I got a different answer. I think Professor Biezen missing the L/2 for the 5000kg board.

  • @riccoventerea5111
    @riccoventerea5111 3 года назад

    Why isn't there a torque produced by the tension holding the mass? Is it because we are assuming the cable is massless?

    • @MichelvanBiezen
      @MichelvanBiezen  3 года назад +1

      We are only considering forces acting on the system, not forces within the system

    • @riccoventerea5111
      @riccoventerea5111 3 года назад

      @@MichelvanBiezen I see, thank you!

  • @wilhelmnantana9547
    @wilhelmnantana9547 9 лет назад +3

    If you cancel the L which is over 2 is it not going to be half the cos of theta

    • @MichelvanBiezen
      @MichelvanBiezen  9 лет назад

      Wilhelm,
      Yes you are correct. I forgot to carry the 1/2 (see below)

    • @otte070
      @otte070 9 лет назад

      Yea I was confused about that as well :)

  • @karimkhan1312
    @karimkhan1312 8 лет назад +1

    very well explained= thank u sir

  • @eugenechoi9146
    @eugenechoi9146 6 лет назад +1

    how come we dont use the perpendicular components of force in this problem? arent we supposed to use mgcostheta?

    • @MichelvanBiezen
      @MichelvanBiezen  6 лет назад +1

      There are two way to solve a problem like this. Using mgcostheta is one of them.

  • @user-nk9fx6zn2l
    @user-nk9fx6zn2l 6 лет назад +1

    Michel, Thank. You.

  • @adeyemirasheed2855
    @adeyemirasheed2855 7 лет назад +1

    great work prof

  • @petermaroney3625
    @petermaroney3625 6 лет назад +1

    Great video! Is another way to go about solving this problem? If you solved for all of the perpendicular forces and d3?

    • @MichelvanBiezen
      @MichelvanBiezen  6 лет назад

      There are often multiple ways to solve these types of problems.

  • @imannajwa4791
    @imannajwa4791 6 лет назад +2

    I tried doing it a couple of times but why am I still getting 120 606.4 instead of 120 483.4??? Anyone?

    • @MichelvanBiezen
      @MichelvanBiezen  6 лет назад +2

      Look at the comments below.

    • @imannajwa4791
      @imannajwa4791 6 лет назад +2

      oh! i think i know where went wrong. i used 9.81 for gravity instead of 9.8 (it made so much difference) but we can use 9.81 for gravity too right?

    • @MichelvanBiezen
      @MichelvanBiezen  6 лет назад +2

      It doesn't matter. g = 9.81 m/sec^2 at the poles and g = 9.78 m/sec^2 at the equator.

    • @imannajwa4791
      @imannajwa4791 6 лет назад +2

      i see. thank u professor! you've help so much and your videos are GREAT!

  • @JohnDoe-kv4ef
    @JohnDoe-kv4ef 7 лет назад +2

    Hey Michel, in this situation, how would one calculate the reaction force at the pivot point?
    Would you split the tension into its two components?

    • @MichelvanBiezen
      @MichelvanBiezen  7 лет назад

      Yes, you will have to find all the x and y components of each force including the tension.

    • @MichelvanBiezen
      @MichelvanBiezen  7 лет назад

      Sometimes you can also find the reaction forces by picking other pivot points and calculating the sum of the torques.

    • @JohnDoe-kv4ef
      @JohnDoe-kv4ef 7 лет назад

      Thanks for the response. I'm assuming that if I were to want to find the x and y components of the tension, I would use the tension as my hypotenuse but what would my angle be?

  • @JesusPerez-ue4dw
    @JesusPerez-ue4dw 9 лет назад

    great videos only that i calculate my force vectors in terms of sines and cosines rather than changing the length, i end up with sines in my numerator and sin 60 in my denominator, im not sure if its better to go youre route

    • @MichelvanBiezen
      @MichelvanBiezen  9 лет назад +1

      Jesus Perez
      In the end everyone finds their favorite method. There are usually different ways to do a problem.
      And you should pick the method that works best for you.
      The methods that I use were chosen from experience with students. These methods seem to be the easiest understood by most.

  • @ibanbista00
    @ibanbista00 8 лет назад +2

    what about 1/2 on (L/2)......after cancellation..

    • @MichelvanBiezen
      @MichelvanBiezen  8 лет назад +1

      +ishwor bista
      Yes, there is an error. (see below)

  • @shahed0098
    @shahed0098 3 года назад +1

    why didnt you take the tension in the other wire? the one between m and M?

    • @MichelvanBiezen
      @MichelvanBiezen  3 года назад +1

      That tension is equal to the weight of m, T = mg

    • @shahed0098
      @shahed0098 3 года назад

      @@MichelvanBiezen thank you!!

  • @TinCarpter
    @TinCarpter 10 лет назад

    Great problem, and you did forget the 1/2 for Mg*cos theta.

    • @MichelvanBiezen
      @MichelvanBiezen  10 лет назад +1

      Thank you and yes, one of my students pointed that out already.

  • @10jconway
    @10jconway 7 лет назад +1

    for the final answer i got 12060N?

  • @94hiralpatel
    @94hiralpatel 10 лет назад

    you are the best!!!!!!!!!!!!!!!!!!!!!!!!!!!!!!!!!!!!!!!!

  • @nurrizzatimohammadzambree9553
    @nurrizzatimohammadzambree9553 2 года назад +1

    I would like to ask why the tension supporting the weight is not taken into account for this calculation,, hi professor...

  • @soymaxxing
    @soymaxxing 9 лет назад

    When you were breaking the down the d3 triangle why did u not use the top right hand corner angle (15 degrees) and not the angle? (which ended up being 75 degrees?

    • @MichelvanBiezen
      @MichelvanBiezen  9 лет назад +1

      Reagan,
      You always want to break it down to a right triangle, where you know the hypotenuse or one of the sides as well as 1 angle.

    • @Omooloola
      @Omooloola 2 года назад

      @@MichelvanBiezen Please, I need your help in understanding, while that angle (the 15°) is the difference of the other two angles (45° and 30°). What rule or theorem states that? Please, I need to learn it, bcos I don't understand. Of course you can also give an explanation with your response. Thanks

  • @ChuaShaoCong
    @ChuaShaoCong 9 лет назад

    I thought the answer is dependent on the lowest no. of significant figures for multiplication and division. Shouldn't the answer be just 1 significant figure since there is the lowest sig fig is 1?

    • @MichelvanBiezen
      @MichelvanBiezen  9 лет назад

      Chua,
      The emphasis here is not significant figures but the understanding of the concepts.
      The numbers in this problem were just randomly chosen with disregard to the significant figures.
      But since you asked, the question now becomes: does the quantity "10" have one or two significant figures? unfortunately, it depends on who you ask. Technically speaking it has 2.

    • @ChuaShaoCong
      @ChuaShaoCong 9 лет назад

      Frankly I am just clarifying so that I could make sense of it. Was the rule that I stated correct? Thanks for taking the time to reply as well, your videos do help:)

  • @janayool1001
    @janayool1001 10 лет назад

    can i find distance d1=sin45*l (theta =45 and we have theta=90 the third theta 45)
    for d2=sin45*l/2
    for d3=sin 15 *l right ?

    • @MichelvanBiezen
      @MichelvanBiezen  10 лет назад

      Jana,
      sin(45) = cos (45)
      However, based on the sketch, you should use cos(45)

    • @janayool1001
      @janayool1001 10 лет назад

      sin 15 not = cos 15

    • @MichelvanBiezen
      @MichelvanBiezen  10 лет назад

      jana yool
      For d3, sin(15) L is correct.

  • @Mason-nz9iw
    @Mason-nz9iw 7 лет назад +1

    Thank you

  • @vfgrfbgrb
    @vfgrfbgrb 5 лет назад

    Michael van what type of triangle is d3 also isnt hyptonuse the longest side

    • @MichelvanBiezen
      @MichelvanBiezen  5 лет назад

      d3 is a distance (not a triangle) It is the perpendicular distance from the line of action of the tension to the pivot point.

  • @phoepyonchoe7876
    @phoepyonchoe7876 7 лет назад +1

    Thank you Sir

  • @tamazouztemaghni6209
    @tamazouztemaghni6209 4 года назад

    i didn't understand why did you cancel the length?

  • @anujpahade3168
    @anujpahade3168 10 лет назад +1

    Sir my ans comes out to be 124736N .......i guess u forgot to take that1/2....overall great video.....

  • @HDPhenomenon
    @HDPhenomenon 5 лет назад

    what about when they dont provide the length?

    • @MichelvanBiezen
      @MichelvanBiezen  5 лет назад

      If the length of the beam is not given, you cannot calculate the torque.

  • @seer9293
    @seer9293 7 лет назад +1

    shouldn't d2 be mg l/2 cos theta1 and not theta2?

  • @aarushichitagi3926
    @aarushichitagi3926 4 года назад

    why isn't the normal force acting on the beam considered?

    • @MichelvanBiezen
      @MichelvanBiezen  4 года назад

      Any forces who's line of action goes through the pivot point can be ignored as they don't produce a torque

  • @hwan00ju
    @hwan00ju 3 года назад

    Hello sir. I didn't heard clearly how you derived the sin 15° :(((

    • @MichelvanBiezen
      @MichelvanBiezen  3 года назад +1

      The 15 degrees = 45 degrees - 30 degrees in this case. To find the torque caused by the cable we take L cos (90 - 15) = L cos (75) = L sin (15)

    • @hwan00ju
      @hwan00ju 3 года назад

      @@MichelvanBiezen thankyou sir! 😭💕

  • @mcmbasaguz2666
    @mcmbasaguz2666 10 лет назад

    thank you you helped me

  • @ptyptypty3
    @ptyptypty3 9 лет назад +1

    Howdy Michel ... (is your first name pronounced like MICHAEL or like Michelle?) .. .. I was wondering what the FORCE is on the BOOM as it presses against the Pivot Point.. Would we just use Vector Addition to figure that out?.. Adding the Tension Vector to the (mg) Vector?.... yielding a Resultant Vector = to the Force on the Boom at the Pivot Point? by the way, I get a Kick out of your Wife editing your Videos when she adds comments. . like the HUNGRY Bear video .. you pointed out how your drawing of a bear was Skinny... so she changed the title to the STARVING Bear.. lol .. congrats on a Great series of VIDEOS!!.. I still refer to your Collection of Videos as the VanBiezen School of Engineering!!.. You cover everything... well mostly... Astronomy, Chemistry, Physics.. Algebra, Calculus.. Probablility.... on and on and ... ON!! THANK YOU!!.. like so many of your Fans here... I can agree that your Videos are the BEST!! They are Clear.. Organized... and use plenty of Examples to teach the Fundamentals... Thank you for all that you do.... .. Peace always.... -- Phil

    • @MichelvanBiezen
      @MichelvanBiezen  9 лет назад +2

      Philip Y
      Hi Philip. Still at it I see.
      Most people call me "Mike", but when I grew up I was called Michel (pronounced like Michelle) (Like in the Beatle song: Michelle ma belle......
      These videos have been an interesting journey. Where it will end up I don't know, but we are having fun doing it. It is however a lot of work (on top of the three jobs that I have).
      Thanks for the comment.
      To understand the forces on the boom better, look at the sum of all the forces in the x-direction and the sum of all the forces in the y-direction and they also add up to zero. (find the components of every force)

  • @hamzabaniissa3598
    @hamzabaniissa3598 7 лет назад +1

    i think at min 8;17 you forgot the 1\2 ?

    • @MichelvanBiezen
      @MichelvanBiezen  7 лет назад

      Yes, as others before you, you found the error. I made some corrections. Thanks.

  • @nildolabtic6990
    @nildolabtic6990 9 лет назад +1

    SORRY,, Correct me if i'm wrong i'm lost of the cancellation.....

    • @MichelvanBiezen
      @MichelvanBiezen  9 лет назад

      Nildo,
      Are you referring to eliminating the "L"?
      If so, the method used is dividing both sides of the equation by L.
      Since the left side is equal to 0, 0 divided by L is still 0

    • @nildolabtic6990
      @nildolabtic6990 9 лет назад +1

      Thank you sir,,,

  • @michellepedrazas5850
    @michellepedrazas5850 10 лет назад

    What happens with the Tension of the hanging mass?

    • @MichelvanBiezen
      @MichelvanBiezen  10 лет назад

      Michelle,
      The tension in the cable holding the hanging mass is equal to the weight of the hanging mass = mg

    • @rahultiwari9003
      @rahultiwari9003 9 лет назад

      then what is the difference between the tension we calculated in the problem and the tension which is equal to mg

  • @reyrey389
    @reyrey389 9 лет назад

    Good video, but does the force of friction act to the right of the beam? ( near the beam)
    Also, is there a perpendicular normal force ? Or is at a 45 degree angle since the bream makes a 45 degree angle with the ground . I say this because in you previous video (example 2) , the beam made an angle with a vertical pole , so I assumed it applied a force to the beam at that angle.
    Thank you

    • @MichelvanBiezen
      @MichelvanBiezen  9 лет назад

      There are no friction forces involved here.
      Problems like this one are solved by summing up all the torques. If there is no angular acceleration, the object is stationary, then the sum of all the torques add up to zero.
      Sometimes you also need to sum up all the forces in the x direction and all the forces in the y direction and those sums will be zero as well.
      In this problem, there are three forces, the weight of the beam, the weight of the hanging object, (both act directly downward) and the tension in the cable.
      There are reactionary forces at the location where the beam touches the ground, but they can be ignored if you choose the pivot point at that location.

  • @Lucerozeus
    @Lucerozeus 9 лет назад

    great video.

  • @elisa000
    @elisa000 9 лет назад

    thank you!

  • @rithikamedari92
    @rithikamedari92 5 лет назад

    Do we account normal force too?

    • @MichelvanBiezen
      @MichelvanBiezen  5 лет назад

      Depends on what part of the problem we are solving. We don't use it when we calculate the net torque.

  • @reyrey389
    @reyrey389 9 лет назад

    Near the pivot*

  • @sakherobeidat
    @sakherobeidat 9 лет назад

    sorry but you miss the half in Mgcos(45) *1/2

  • @lanmisu
    @lanmisu 9 лет назад

    he is rock!!!

  • @erikcachicamo
    @erikcachicamo 9 лет назад

    I love you.

  • @huzaifahmed6293
    @huzaifahmed6293 4 года назад

    Angle 15 for d3 , yu took needs little elaboration

  • @MrDoYouWannaBeOnTop
    @MrDoYouWannaBeOnTop 9 лет назад

    how to find the pivot point?

    • @MichelvanBiezen
      @MichelvanBiezen  9 лет назад +1

      +MrDoYouWannaBeOnTop
      You can pick any point to pick the pivot point.
      But typically you want to pick a point that will eliminate one unknown force.

    • @MrDoYouWannaBeOnTop
      @MrDoYouWannaBeOnTop 9 лет назад

      oh okay. thanks!

  • @EsimanzonzoNewsChannel
    @EsimanzonzoNewsChannel 8 лет назад

    there is a error in video

    • @MichelvanBiezen
      @MichelvanBiezen  8 лет назад

      +Nduduzo Mahlobo
      Yes there is, thanks for letting me know.

  • @christianramos6440
    @christianramos6440 11 лет назад

    After canceling the L's you forgot to put the remaining 1/2 that belongs in the Mgcos

  • @olebogengmoremi
    @olebogengmoremi 8 лет назад +3

    I got T= 120 483.44 N Sir . You made a mistake :) , you forgot 1/2

  • @earlrusselllavilla6193
    @earlrusselllavilla6193 7 лет назад +1

    i feel so bad bcos he did not include the 1/2 that should be multiplied with 5000 😂 😯✌

  • @zeinomadikizela4783
    @zeinomadikizela4783 8 лет назад +1

    My answer is 120 483.44N, Did anyone get that?

    • @MichelvanBiezen
      @MichelvanBiezen  8 лет назад +1

      +Zeino Madikizela That is correct. (although too many significant figures)

    • @zeinomadikizela4783
      @zeinomadikizela4783 8 лет назад

      How many significant figures are recommended?

    • @MichelvanBiezen
      @MichelvanBiezen  8 лет назад

      +Zeino Madikizela Typically, it the data is given with 3 significant figures, your answer should only have 3 significant figures. (It depends on the accuracy of the numbers given in the problem).

    • @zeinomadikizela4783
      @zeinomadikizela4783 8 лет назад

      Thanks, totally forgot about taking the least accurate one into account as the number of significant figures.

  • @abdullahahmethan2890
    @abdullahahmethan2890 10 лет назад +1

    T= (gSin(45)*(2500+200))/ sin(15)

    • @TrueYears
      @TrueYears 9 лет назад

      Its 2000kg not 200kg

  • @anilkumarsharma1205
    @anilkumarsharma1205 5 лет назад +1

    are you make an app over crane engineering and how much energy used by crane in running and lifting and efficiency of engine so we isolated the tachometer hours meter time and diesel consumption and various forms of energy in the same time and actual time or real time according to your wrist watch
    this will help any crane engineering staff and operated machine
    pollution solution become easy for millions of trillions of dollars world wide used in making pollution solution

  • @abcdef2069
    @abcdef2069 8 лет назад

    d3 introduction is a downfall which makes things look un-intuitive.
    he should have kept all the sine's which came from the cross product.
    = T * length * sin (165) which is intuitive

  • @zack150
    @zack150 7 лет назад +1

    A slightly toned down gru

  • @obakengatom_sweetbrotherof1081
    @obakengatom_sweetbrotherof1081 7 лет назад +1

    I THINK THE ANSWER IS 12048.3N

  • @ayimmas4672
    @ayimmas4672 8 лет назад

    yup...answer is 120,615

  • @alexanderhenry7718
    @alexanderhenry7718 9 лет назад

    It's never good to use 45 degrees in an example because you can't always be sure what angle in the triangle is being used. Also because sin 45 and cos 45 are the same.